Is the concept of dimension still well defined for non-finite dimensional spaces? [duplicate]












4















This question already has an answer here:




  • Proof that two basis of a vector space have the same cardinality in the infinite-dimensional case

    2 answers





The question is quite simple: if $mathbb{V}$ is a vector space and $B$ and $B'$ are basis for $mathbb V$, then do $B$ and $B'$ have the same cardinality?




I've tried to answer the question as follows: suppose that $B'$ is bigger than $B$. Then there is a map $f:B'to B$ surjective but non-injective. Extending this to a linear map $f':mathbb Vtomathbb V$ we have that $mathbb V/ker f'cong mathrm{Im} f'=mathbb V$, with $ker f'$ being non-trivial. But is this really a contradiction? (of course it is in finite dimension, since we know that all basis have the same cardinality in that case, but this is exactly what we are trying to prove here...)










share|cite|improve this question













marked as duplicate by Asaf Karagila cardinals
Users with the  cardinals badge can single-handedly close cardinals questions as duplicates and reopen them as needed.

StackExchange.ready(function() {
if (StackExchange.options.isMobile) return;

$('.dupe-hammer-message-hover:not(.hover-bound)').each(function() {
var $hover = $(this).addClass('hover-bound'),
$msg = $hover.siblings('.dupe-hammer-message');

$hover.hover(
function() {
$hover.showInfoMessage('', {
messageElement: $msg.clone().show(),
transient: false,
position: { my: 'bottom left', at: 'top center', offsetTop: -7 },
dismissable: false,
relativeToBody: true
});
},
function() {
StackExchange.helpers.removeMessages();
}
);
});
});
Nov 29 at 19:07


This question has been asked before and already has an answer. If those answers do not fully address your question, please ask a new question.















  • I think it's true that $B$ and $B'$ have the same cardinality but I don't think what you did gives a contradiction. I'm too lazy to find a reference. I'm sure someone else will find one
    – mathworker21
    Nov 29 at 18:39






  • 1




    What you give can't possibly be a contradiction as a surjective but non-injective function exists between any two infinite sets. In particular ones of the same cardinality. The argument is much easier though.
    – DRF
    Nov 29 at 18:43
















4















This question already has an answer here:




  • Proof that two basis of a vector space have the same cardinality in the infinite-dimensional case

    2 answers





The question is quite simple: if $mathbb{V}$ is a vector space and $B$ and $B'$ are basis for $mathbb V$, then do $B$ and $B'$ have the same cardinality?




I've tried to answer the question as follows: suppose that $B'$ is bigger than $B$. Then there is a map $f:B'to B$ surjective but non-injective. Extending this to a linear map $f':mathbb Vtomathbb V$ we have that $mathbb V/ker f'cong mathrm{Im} f'=mathbb V$, with $ker f'$ being non-trivial. But is this really a contradiction? (of course it is in finite dimension, since we know that all basis have the same cardinality in that case, but this is exactly what we are trying to prove here...)










share|cite|improve this question













marked as duplicate by Asaf Karagila cardinals
Users with the  cardinals badge can single-handedly close cardinals questions as duplicates and reopen them as needed.

StackExchange.ready(function() {
if (StackExchange.options.isMobile) return;

$('.dupe-hammer-message-hover:not(.hover-bound)').each(function() {
var $hover = $(this).addClass('hover-bound'),
$msg = $hover.siblings('.dupe-hammer-message');

$hover.hover(
function() {
$hover.showInfoMessage('', {
messageElement: $msg.clone().show(),
transient: false,
position: { my: 'bottom left', at: 'top center', offsetTop: -7 },
dismissable: false,
relativeToBody: true
});
},
function() {
StackExchange.helpers.removeMessages();
}
);
});
});
Nov 29 at 19:07


This question has been asked before and already has an answer. If those answers do not fully address your question, please ask a new question.















  • I think it's true that $B$ and $B'$ have the same cardinality but I don't think what you did gives a contradiction. I'm too lazy to find a reference. I'm sure someone else will find one
    – mathworker21
    Nov 29 at 18:39






  • 1




    What you give can't possibly be a contradiction as a surjective but non-injective function exists between any two infinite sets. In particular ones of the same cardinality. The argument is much easier though.
    – DRF
    Nov 29 at 18:43














4












4








4


1






This question already has an answer here:




  • Proof that two basis of a vector space have the same cardinality in the infinite-dimensional case

    2 answers





The question is quite simple: if $mathbb{V}$ is a vector space and $B$ and $B'$ are basis for $mathbb V$, then do $B$ and $B'$ have the same cardinality?




I've tried to answer the question as follows: suppose that $B'$ is bigger than $B$. Then there is a map $f:B'to B$ surjective but non-injective. Extending this to a linear map $f':mathbb Vtomathbb V$ we have that $mathbb V/ker f'cong mathrm{Im} f'=mathbb V$, with $ker f'$ being non-trivial. But is this really a contradiction? (of course it is in finite dimension, since we know that all basis have the same cardinality in that case, but this is exactly what we are trying to prove here...)










share|cite|improve this question














This question already has an answer here:




  • Proof that two basis of a vector space have the same cardinality in the infinite-dimensional case

    2 answers





The question is quite simple: if $mathbb{V}$ is a vector space and $B$ and $B'$ are basis for $mathbb V$, then do $B$ and $B'$ have the same cardinality?




I've tried to answer the question as follows: suppose that $B'$ is bigger than $B$. Then there is a map $f:B'to B$ surjective but non-injective. Extending this to a linear map $f':mathbb Vtomathbb V$ we have that $mathbb V/ker f'cong mathrm{Im} f'=mathbb V$, with $ker f'$ being non-trivial. But is this really a contradiction? (of course it is in finite dimension, since we know that all basis have the same cardinality in that case, but this is exactly what we are trying to prove here...)





This question already has an answer here:




  • Proof that two basis of a vector space have the same cardinality in the infinite-dimensional case

    2 answers








linear-algebra vector-spaces cardinals infinity change-of-basis






share|cite|improve this question













share|cite|improve this question











share|cite|improve this question




share|cite|improve this question










asked Nov 29 at 18:37









Anderson Felipe Viveiros

1,098727




1,098727




marked as duplicate by Asaf Karagila cardinals
Users with the  cardinals badge can single-handedly close cardinals questions as duplicates and reopen them as needed.

StackExchange.ready(function() {
if (StackExchange.options.isMobile) return;

$('.dupe-hammer-message-hover:not(.hover-bound)').each(function() {
var $hover = $(this).addClass('hover-bound'),
$msg = $hover.siblings('.dupe-hammer-message');

$hover.hover(
function() {
$hover.showInfoMessage('', {
messageElement: $msg.clone().show(),
transient: false,
position: { my: 'bottom left', at: 'top center', offsetTop: -7 },
dismissable: false,
relativeToBody: true
});
},
function() {
StackExchange.helpers.removeMessages();
}
);
});
});
Nov 29 at 19:07


This question has been asked before and already has an answer. If those answers do not fully address your question, please ask a new question.






marked as duplicate by Asaf Karagila cardinals
Users with the  cardinals badge can single-handedly close cardinals questions as duplicates and reopen them as needed.

StackExchange.ready(function() {
if (StackExchange.options.isMobile) return;

$('.dupe-hammer-message-hover:not(.hover-bound)').each(function() {
var $hover = $(this).addClass('hover-bound'),
$msg = $hover.siblings('.dupe-hammer-message');

$hover.hover(
function() {
$hover.showInfoMessage('', {
messageElement: $msg.clone().show(),
transient: false,
position: { my: 'bottom left', at: 'top center', offsetTop: -7 },
dismissable: false,
relativeToBody: true
});
},
function() {
StackExchange.helpers.removeMessages();
}
);
});
});
Nov 29 at 19:07


This question has been asked before and already has an answer. If those answers do not fully address your question, please ask a new question.














  • I think it's true that $B$ and $B'$ have the same cardinality but I don't think what you did gives a contradiction. I'm too lazy to find a reference. I'm sure someone else will find one
    – mathworker21
    Nov 29 at 18:39






  • 1




    What you give can't possibly be a contradiction as a surjective but non-injective function exists between any two infinite sets. In particular ones of the same cardinality. The argument is much easier though.
    – DRF
    Nov 29 at 18:43


















  • I think it's true that $B$ and $B'$ have the same cardinality but I don't think what you did gives a contradiction. I'm too lazy to find a reference. I'm sure someone else will find one
    – mathworker21
    Nov 29 at 18:39






  • 1




    What you give can't possibly be a contradiction as a surjective but non-injective function exists between any two infinite sets. In particular ones of the same cardinality. The argument is much easier though.
    – DRF
    Nov 29 at 18:43
















I think it's true that $B$ and $B'$ have the same cardinality but I don't think what you did gives a contradiction. I'm too lazy to find a reference. I'm sure someone else will find one
– mathworker21
Nov 29 at 18:39




I think it's true that $B$ and $B'$ have the same cardinality but I don't think what you did gives a contradiction. I'm too lazy to find a reference. I'm sure someone else will find one
– mathworker21
Nov 29 at 18:39




1




1




What you give can't possibly be a contradiction as a surjective but non-injective function exists between any two infinite sets. In particular ones of the same cardinality. The argument is much easier though.
– DRF
Nov 29 at 18:43




What you give can't possibly be a contradiction as a surjective but non-injective function exists between any two infinite sets. In particular ones of the same cardinality. The argument is much easier though.
– DRF
Nov 29 at 18:43










1 Answer
1






active

oldest

votes


















2














Your argument doesn't work, since - as mentioned in the comments - given any infinite set $A$ there is a map $f:Arightarrow A$ which is surjective but not injective (or injective but not surjective, if you prefer).



However, the statement is true, and the proof is simpler:




  • Suppose $B, B'$ are infinite bases of $mathbb{V}$ with $vert Bvert<vert B'vert$.


  • For each $bin B$, there is a finite $F_bsubseteq B'$ with $binlangle F_brangle$.


  • Since the union of $vert Bvert$-many finite sets has cardinality $vert Bvert$, the set $$hat{B}=bigcup_{bin B}F_b$$ has cardinality $vert Bvert$.


  • Since $vert Bvert<vert B'vert$, there must therefore be some $ain B'setminushat{B}$.


  • But since $B$ spans $mathbb{V}$, $a$ can be written as a linear combination of elements of $B$, and hence can be written as a linear combination of elements of $hat{B}$. This contradicts the linear independence of $B'$.





As a quick pedagogical aside, here's an attempted proof which doesn't quite work:



Since $B$ is a basis for $mathbb{V}$, we have that every element of $B'$ can be represented as a finite linear combination of elements of the field of scalars $F$. Using the fact that the set of finite subsets of an infinite set has the same cardinality as the original set, the cardinality of the set of linear combinations of elements of $B$ is $vert Bverttimesvert Fvert$, and so we get $$vert B'vertlevert Bverttimesvert Fvert.$$



In case $F$ is "small" - that is, in case $vert Fvertlevert Bvert$ - then we get $vert B'vertlevert Bvert$ (since multiplication of two infinite cardinalities just results in the larger of the two). But in case $F$ is large, we don't get anything. E.g. this argument doesn't rule out the possibility of a vector space over a field of cardinality $aleph_{omega^3+omegacdot 842+17}$ with one basis of cardinality $aleph_{omega^2+9}$ and another basis of cardinality $aleph_0$.





Note that we've used the axiom of choice crucially in the above when we calculated the cardinality of $hat{B}$ (which we needed in order to conclude the existence of $a$). Without the axiom of choice, the argument above breaks down. This is the only essential use of choice - note that the second bulletpoint does not require choice, since there is a unique minimal choice of $F_b$.



The full axiom of choice is not needed, and the general study of how much choice is needed to prove various mathematical results - around vector spaces and other topics - is quite rich; I just want to point out the reliance on something beyond set theory without choice, here.






share|cite|improve this answer























  • +1 Very nice. Was writing it up but got sidetracked by the size of the underlying field.
    – DRF
    Nov 29 at 19:03










  • @DRF Yes, if you just calculate the size of the set of all linear combinations from $B$ you only get the result in case the field of scalars is no larger than the smaller basis.
    – Noah Schweber
    Nov 29 at 19:06












  • Since you bring up choice, this is really a compactness argument, so perhaps unsurprisingly, it only requires the Boolean Prime Ideal theorem, and indeed it is weaker.
    – Asaf Karagila
    Nov 29 at 19:09










  • @AsafKaragila Yes, I didn't mean to imply it needed full choice - edited!
    – Noah Schweber
    Nov 29 at 19:10










  • Just like you shouldn't write in your CV that a paper was submitted, until you actually submitted it, you shouldn't write in your comment that you've edited your answer until you've done that. It's all kinds of confusing! :-)
    – Asaf Karagila
    Nov 29 at 19:11


















1 Answer
1






active

oldest

votes








1 Answer
1






active

oldest

votes









active

oldest

votes






active

oldest

votes









2














Your argument doesn't work, since - as mentioned in the comments - given any infinite set $A$ there is a map $f:Arightarrow A$ which is surjective but not injective (or injective but not surjective, if you prefer).



However, the statement is true, and the proof is simpler:




  • Suppose $B, B'$ are infinite bases of $mathbb{V}$ with $vert Bvert<vert B'vert$.


  • For each $bin B$, there is a finite $F_bsubseteq B'$ with $binlangle F_brangle$.


  • Since the union of $vert Bvert$-many finite sets has cardinality $vert Bvert$, the set $$hat{B}=bigcup_{bin B}F_b$$ has cardinality $vert Bvert$.


  • Since $vert Bvert<vert B'vert$, there must therefore be some $ain B'setminushat{B}$.


  • But since $B$ spans $mathbb{V}$, $a$ can be written as a linear combination of elements of $B$, and hence can be written as a linear combination of elements of $hat{B}$. This contradicts the linear independence of $B'$.





As a quick pedagogical aside, here's an attempted proof which doesn't quite work:



Since $B$ is a basis for $mathbb{V}$, we have that every element of $B'$ can be represented as a finite linear combination of elements of the field of scalars $F$. Using the fact that the set of finite subsets of an infinite set has the same cardinality as the original set, the cardinality of the set of linear combinations of elements of $B$ is $vert Bverttimesvert Fvert$, and so we get $$vert B'vertlevert Bverttimesvert Fvert.$$



In case $F$ is "small" - that is, in case $vert Fvertlevert Bvert$ - then we get $vert B'vertlevert Bvert$ (since multiplication of two infinite cardinalities just results in the larger of the two). But in case $F$ is large, we don't get anything. E.g. this argument doesn't rule out the possibility of a vector space over a field of cardinality $aleph_{omega^3+omegacdot 842+17}$ with one basis of cardinality $aleph_{omega^2+9}$ and another basis of cardinality $aleph_0$.





Note that we've used the axiom of choice crucially in the above when we calculated the cardinality of $hat{B}$ (which we needed in order to conclude the existence of $a$). Without the axiom of choice, the argument above breaks down. This is the only essential use of choice - note that the second bulletpoint does not require choice, since there is a unique minimal choice of $F_b$.



The full axiom of choice is not needed, and the general study of how much choice is needed to prove various mathematical results - around vector spaces and other topics - is quite rich; I just want to point out the reliance on something beyond set theory without choice, here.






share|cite|improve this answer























  • +1 Very nice. Was writing it up but got sidetracked by the size of the underlying field.
    – DRF
    Nov 29 at 19:03










  • @DRF Yes, if you just calculate the size of the set of all linear combinations from $B$ you only get the result in case the field of scalars is no larger than the smaller basis.
    – Noah Schweber
    Nov 29 at 19:06












  • Since you bring up choice, this is really a compactness argument, so perhaps unsurprisingly, it only requires the Boolean Prime Ideal theorem, and indeed it is weaker.
    – Asaf Karagila
    Nov 29 at 19:09










  • @AsafKaragila Yes, I didn't mean to imply it needed full choice - edited!
    – Noah Schweber
    Nov 29 at 19:10










  • Just like you shouldn't write in your CV that a paper was submitted, until you actually submitted it, you shouldn't write in your comment that you've edited your answer until you've done that. It's all kinds of confusing! :-)
    – Asaf Karagila
    Nov 29 at 19:11
















2














Your argument doesn't work, since - as mentioned in the comments - given any infinite set $A$ there is a map $f:Arightarrow A$ which is surjective but not injective (or injective but not surjective, if you prefer).



However, the statement is true, and the proof is simpler:




  • Suppose $B, B'$ are infinite bases of $mathbb{V}$ with $vert Bvert<vert B'vert$.


  • For each $bin B$, there is a finite $F_bsubseteq B'$ with $binlangle F_brangle$.


  • Since the union of $vert Bvert$-many finite sets has cardinality $vert Bvert$, the set $$hat{B}=bigcup_{bin B}F_b$$ has cardinality $vert Bvert$.


  • Since $vert Bvert<vert B'vert$, there must therefore be some $ain B'setminushat{B}$.


  • But since $B$ spans $mathbb{V}$, $a$ can be written as a linear combination of elements of $B$, and hence can be written as a linear combination of elements of $hat{B}$. This contradicts the linear independence of $B'$.





As a quick pedagogical aside, here's an attempted proof which doesn't quite work:



Since $B$ is a basis for $mathbb{V}$, we have that every element of $B'$ can be represented as a finite linear combination of elements of the field of scalars $F$. Using the fact that the set of finite subsets of an infinite set has the same cardinality as the original set, the cardinality of the set of linear combinations of elements of $B$ is $vert Bverttimesvert Fvert$, and so we get $$vert B'vertlevert Bverttimesvert Fvert.$$



In case $F$ is "small" - that is, in case $vert Fvertlevert Bvert$ - then we get $vert B'vertlevert Bvert$ (since multiplication of two infinite cardinalities just results in the larger of the two). But in case $F$ is large, we don't get anything. E.g. this argument doesn't rule out the possibility of a vector space over a field of cardinality $aleph_{omega^3+omegacdot 842+17}$ with one basis of cardinality $aleph_{omega^2+9}$ and another basis of cardinality $aleph_0$.





Note that we've used the axiom of choice crucially in the above when we calculated the cardinality of $hat{B}$ (which we needed in order to conclude the existence of $a$). Without the axiom of choice, the argument above breaks down. This is the only essential use of choice - note that the second bulletpoint does not require choice, since there is a unique minimal choice of $F_b$.



The full axiom of choice is not needed, and the general study of how much choice is needed to prove various mathematical results - around vector spaces and other topics - is quite rich; I just want to point out the reliance on something beyond set theory without choice, here.






share|cite|improve this answer























  • +1 Very nice. Was writing it up but got sidetracked by the size of the underlying field.
    – DRF
    Nov 29 at 19:03










  • @DRF Yes, if you just calculate the size of the set of all linear combinations from $B$ you only get the result in case the field of scalars is no larger than the smaller basis.
    – Noah Schweber
    Nov 29 at 19:06












  • Since you bring up choice, this is really a compactness argument, so perhaps unsurprisingly, it only requires the Boolean Prime Ideal theorem, and indeed it is weaker.
    – Asaf Karagila
    Nov 29 at 19:09










  • @AsafKaragila Yes, I didn't mean to imply it needed full choice - edited!
    – Noah Schweber
    Nov 29 at 19:10










  • Just like you shouldn't write in your CV that a paper was submitted, until you actually submitted it, you shouldn't write in your comment that you've edited your answer until you've done that. It's all kinds of confusing! :-)
    – Asaf Karagila
    Nov 29 at 19:11














2












2








2






Your argument doesn't work, since - as mentioned in the comments - given any infinite set $A$ there is a map $f:Arightarrow A$ which is surjective but not injective (or injective but not surjective, if you prefer).



However, the statement is true, and the proof is simpler:




  • Suppose $B, B'$ are infinite bases of $mathbb{V}$ with $vert Bvert<vert B'vert$.


  • For each $bin B$, there is a finite $F_bsubseteq B'$ with $binlangle F_brangle$.


  • Since the union of $vert Bvert$-many finite sets has cardinality $vert Bvert$, the set $$hat{B}=bigcup_{bin B}F_b$$ has cardinality $vert Bvert$.


  • Since $vert Bvert<vert B'vert$, there must therefore be some $ain B'setminushat{B}$.


  • But since $B$ spans $mathbb{V}$, $a$ can be written as a linear combination of elements of $B$, and hence can be written as a linear combination of elements of $hat{B}$. This contradicts the linear independence of $B'$.





As a quick pedagogical aside, here's an attempted proof which doesn't quite work:



Since $B$ is a basis for $mathbb{V}$, we have that every element of $B'$ can be represented as a finite linear combination of elements of the field of scalars $F$. Using the fact that the set of finite subsets of an infinite set has the same cardinality as the original set, the cardinality of the set of linear combinations of elements of $B$ is $vert Bverttimesvert Fvert$, and so we get $$vert B'vertlevert Bverttimesvert Fvert.$$



In case $F$ is "small" - that is, in case $vert Fvertlevert Bvert$ - then we get $vert B'vertlevert Bvert$ (since multiplication of two infinite cardinalities just results in the larger of the two). But in case $F$ is large, we don't get anything. E.g. this argument doesn't rule out the possibility of a vector space over a field of cardinality $aleph_{omega^3+omegacdot 842+17}$ with one basis of cardinality $aleph_{omega^2+9}$ and another basis of cardinality $aleph_0$.





Note that we've used the axiom of choice crucially in the above when we calculated the cardinality of $hat{B}$ (which we needed in order to conclude the existence of $a$). Without the axiom of choice, the argument above breaks down. This is the only essential use of choice - note that the second bulletpoint does not require choice, since there is a unique minimal choice of $F_b$.



The full axiom of choice is not needed, and the general study of how much choice is needed to prove various mathematical results - around vector spaces and other topics - is quite rich; I just want to point out the reliance on something beyond set theory without choice, here.






share|cite|improve this answer














Your argument doesn't work, since - as mentioned in the comments - given any infinite set $A$ there is a map $f:Arightarrow A$ which is surjective but not injective (or injective but not surjective, if you prefer).



However, the statement is true, and the proof is simpler:




  • Suppose $B, B'$ are infinite bases of $mathbb{V}$ with $vert Bvert<vert B'vert$.


  • For each $bin B$, there is a finite $F_bsubseteq B'$ with $binlangle F_brangle$.


  • Since the union of $vert Bvert$-many finite sets has cardinality $vert Bvert$, the set $$hat{B}=bigcup_{bin B}F_b$$ has cardinality $vert Bvert$.


  • Since $vert Bvert<vert B'vert$, there must therefore be some $ain B'setminushat{B}$.


  • But since $B$ spans $mathbb{V}$, $a$ can be written as a linear combination of elements of $B$, and hence can be written as a linear combination of elements of $hat{B}$. This contradicts the linear independence of $B'$.





As a quick pedagogical aside, here's an attempted proof which doesn't quite work:



Since $B$ is a basis for $mathbb{V}$, we have that every element of $B'$ can be represented as a finite linear combination of elements of the field of scalars $F$. Using the fact that the set of finite subsets of an infinite set has the same cardinality as the original set, the cardinality of the set of linear combinations of elements of $B$ is $vert Bverttimesvert Fvert$, and so we get $$vert B'vertlevert Bverttimesvert Fvert.$$



In case $F$ is "small" - that is, in case $vert Fvertlevert Bvert$ - then we get $vert B'vertlevert Bvert$ (since multiplication of two infinite cardinalities just results in the larger of the two). But in case $F$ is large, we don't get anything. E.g. this argument doesn't rule out the possibility of a vector space over a field of cardinality $aleph_{omega^3+omegacdot 842+17}$ with one basis of cardinality $aleph_{omega^2+9}$ and another basis of cardinality $aleph_0$.





Note that we've used the axiom of choice crucially in the above when we calculated the cardinality of $hat{B}$ (which we needed in order to conclude the existence of $a$). Without the axiom of choice, the argument above breaks down. This is the only essential use of choice - note that the second bulletpoint does not require choice, since there is a unique minimal choice of $F_b$.



The full axiom of choice is not needed, and the general study of how much choice is needed to prove various mathematical results - around vector spaces and other topics - is quite rich; I just want to point out the reliance on something beyond set theory without choice, here.







share|cite|improve this answer














share|cite|improve this answer



share|cite|improve this answer








edited Nov 29 at 19:12

























answered Nov 29 at 18:57









Noah Schweber

120k10146279




120k10146279












  • +1 Very nice. Was writing it up but got sidetracked by the size of the underlying field.
    – DRF
    Nov 29 at 19:03










  • @DRF Yes, if you just calculate the size of the set of all linear combinations from $B$ you only get the result in case the field of scalars is no larger than the smaller basis.
    – Noah Schweber
    Nov 29 at 19:06












  • Since you bring up choice, this is really a compactness argument, so perhaps unsurprisingly, it only requires the Boolean Prime Ideal theorem, and indeed it is weaker.
    – Asaf Karagila
    Nov 29 at 19:09










  • @AsafKaragila Yes, I didn't mean to imply it needed full choice - edited!
    – Noah Schweber
    Nov 29 at 19:10










  • Just like you shouldn't write in your CV that a paper was submitted, until you actually submitted it, you shouldn't write in your comment that you've edited your answer until you've done that. It's all kinds of confusing! :-)
    – Asaf Karagila
    Nov 29 at 19:11


















  • +1 Very nice. Was writing it up but got sidetracked by the size of the underlying field.
    – DRF
    Nov 29 at 19:03










  • @DRF Yes, if you just calculate the size of the set of all linear combinations from $B$ you only get the result in case the field of scalars is no larger than the smaller basis.
    – Noah Schweber
    Nov 29 at 19:06












  • Since you bring up choice, this is really a compactness argument, so perhaps unsurprisingly, it only requires the Boolean Prime Ideal theorem, and indeed it is weaker.
    – Asaf Karagila
    Nov 29 at 19:09










  • @AsafKaragila Yes, I didn't mean to imply it needed full choice - edited!
    – Noah Schweber
    Nov 29 at 19:10










  • Just like you shouldn't write in your CV that a paper was submitted, until you actually submitted it, you shouldn't write in your comment that you've edited your answer until you've done that. It's all kinds of confusing! :-)
    – Asaf Karagila
    Nov 29 at 19:11
















+1 Very nice. Was writing it up but got sidetracked by the size of the underlying field.
– DRF
Nov 29 at 19:03




+1 Very nice. Was writing it up but got sidetracked by the size of the underlying field.
– DRF
Nov 29 at 19:03












@DRF Yes, if you just calculate the size of the set of all linear combinations from $B$ you only get the result in case the field of scalars is no larger than the smaller basis.
– Noah Schweber
Nov 29 at 19:06






@DRF Yes, if you just calculate the size of the set of all linear combinations from $B$ you only get the result in case the field of scalars is no larger than the smaller basis.
– Noah Schweber
Nov 29 at 19:06














Since you bring up choice, this is really a compactness argument, so perhaps unsurprisingly, it only requires the Boolean Prime Ideal theorem, and indeed it is weaker.
– Asaf Karagila
Nov 29 at 19:09




Since you bring up choice, this is really a compactness argument, so perhaps unsurprisingly, it only requires the Boolean Prime Ideal theorem, and indeed it is weaker.
– Asaf Karagila
Nov 29 at 19:09












@AsafKaragila Yes, I didn't mean to imply it needed full choice - edited!
– Noah Schweber
Nov 29 at 19:10




@AsafKaragila Yes, I didn't mean to imply it needed full choice - edited!
– Noah Schweber
Nov 29 at 19:10












Just like you shouldn't write in your CV that a paper was submitted, until you actually submitted it, you shouldn't write in your comment that you've edited your answer until you've done that. It's all kinds of confusing! :-)
– Asaf Karagila
Nov 29 at 19:11




Just like you shouldn't write in your CV that a paper was submitted, until you actually submitted it, you shouldn't write in your comment that you've edited your answer until you've done that. It's all kinds of confusing! :-)
– Asaf Karagila
Nov 29 at 19:11



Popular posts from this blog

To store a contact into the json file from server.js file using a class in NodeJS

Redirect URL with Chrome Remote Debugging Android Devices

Dieringhausen